What two times could this be on the 24-hour clock?

What Two Times Could This Be On The 24-hour Clock?

Answers

Answer 1
I think it’s 1:40 am or 1:40 pm

Related Questions

Which system of linear inequalities is represented by
the graph?
Oy> x-2 and y < x + 1
O y< x-2 and y > x + 1
Oy x + 1
O y > x-2 and y < x + 1

Answers

Answer:

The correct option is;

y < x - 2, and y > x + 1

Step-by-step explanation:

The given graph of inequalities is made up of parallel lines. Therefore, the slope of the inequalities are equal

By examination of the graph, the common slope = (Increase in y-value)/(Corresponding increase in x-value) = (0 - 1)/(-1 - 0) = 1

Therefore, the slope = 1

We note that the there are three different colored regions, therefore, the different colored regions opposite to each inequalities should be the areas of interest

The y-intercept for the upper bounding linear inequality, (y >) is 1

The y-intercept for the lower bounding linear inequality, (y <) is -2

The two inequalities are y > x + 1 and y < x - 2

The correct option is y < x - 2, and y > x + 1.

The system of linear inequalities is represented by  the graph is y> x-2 and y < x + 1

Inequalities is an expression that shows the non equal comparison of two or more variables and numbers.

Given that:

y and x are variables, plotting the inequalities using geogebra online graphing tool.

The system of linear inequalities is represented by  the graph is y> x-2 and y < x + 1

Find out more on linear inequalities at: https://brainly.com/question/21103162

What is 12.5% of 72

Answers

Answer:

[tex]\boxed{9}[/tex]

Step-by-step explanation:

[tex]\sf of \ refers \ to \ multiplication.[/tex]

[tex]12.5\% \times 72[/tex]

[tex]\frac{12.5}{100} \times 72[/tex]

[tex]\sf Multiply.[/tex]

[tex]\frac{900}{100} =9[/tex]

20 squared (+5) divided by 100

Answers

The answer is 4.05

Step-by-step explanation:

20^2 is 20•20 which is 400 || +5=405 || /100=4.05

Find the measure of each side indicated. Round to the nearest tenth.

A) 19.8
C) 24.9
B) 27.2
D) 25.3

Answers

Answer:

D. 25.3

Step-by-step explanation:

tan∅ = opposite over adjacent

Step 1: Write equation

tan66.5° = x/11

Step 2: Multiply both sides by 11

11tan66.5° = x

Step 3: Evaluate

x = 25.2983

x ≈ 25.3

Answer:

[tex]\huge\boxed{x = 25.3}[/tex]

Step-by-step explanation:

Tan θ = opposite / adjacent

Where θ = 66.5 , opposite = x and adjacent = 11

Tan 66.5 = x / 11

2.3 * 11 = x

25.3 = x

OR

x = 25.3

1.Solve by factorization method: x+1/x=11 1/11 2.Comment on the nature of roots for 4x^2-5=2(〖x+1)〗^2-7 plz, help...

Answers

Answer:

The equation

[tex]4\,x^2-5=2\,(x+1)^2-7[/tex]

can be solved by first expanding all indicated operations, and later when the constant terms disappear, by factoring out 2x , leaving the equation as a product of two factors equal zero, from which it is easy to extract the roots. See below.

Step-by-step explanation:

When solving for x in the following expression, and using factoring to apply at the end the zero product theorem:

[tex]4\,x^2-5=2\,(x+1)^2-7\\4\,x^2-5=2\,(x^2+2x+1)-7\\4\,x^2-5=2\,x^2+4\,x+2-7\\4\,x^2-5=2\.x^2+4\,x-5\\4\,x^2=2\,x^2+4\,x\\4\,x^2-2\,x^2-4\,x=0\\2\,x^2-4\,x=0\\2\,x\,(x-2)=0[/tex]

We observe that for the last product, to get a zero, x has to be zero (making the first factor zero), or x has to be "2" making the binomial factor zero.

1)Sheyna drive to the lake and back. It took two hours less time to get there than it did to get back. The average speed on the trip there was 60 mph. The average speed on the way back was 36 mph. How many hours did the trip there take?

Answers

Answer:

8 hours

Step-by-step explanation:

Given:

Sheyna drives to the lake with average speed of 60 mph and

[tex]v_1 = 60\ mph[/tex]

Sheyna drives back from the lake with average speed of 36 mph

[tex]v_2 = 36\ mph[/tex]

It took 2 hours less time to get there than it did to get back.

Let [tex]t_1[/tex] be the time taken to drive to lake.

Let [tex]t_2[/tex] be the time taken to drive back from lake.

[tex]t_2-t_1 = 2[/tex] hrs ..... (1)

To find:

Total time taken = ?

[tex]t_1+t_2 = ?[/tex]

Solution:

Let D be the distance to lake.

Formula for time is given as:

[tex]Time =\dfrac{Distance}{Speed }[/tex]

[tex]t_1 = \dfrac{D}{60}\ hrs[/tex]

[tex]t_2 = \dfrac{D}{36}\ hrs[/tex]

Putting in equation (1):

[tex]\dfrac{D}{36}-\dfrac{D}{60} = 2\\\Rightarrow \dfrac{5D-3D}{180} = 2\\\Rightarrow \dfrac{2D}{180} = 2\\\Rightarrow D = 180\ miles[/tex]

So,

[tex]t_1 = \dfrac{180}{60}\ hrs = 3 \ hrs[/tex]

[tex]t_2 = \dfrac{180}{36}\ hrs = 5\ hrs[/tex]

So, the answer is:

[tex]t_1+t_2 = \bold{8\ hrs}[/tex]

A shop sells DVDs and CDs.

DVDs are sold at one price.
CDs are sold at a different price.

2 DVDs and 1 CD cost £35
2 DVDs and 2 CDs cost £45

Martin has £50 Does he have enough to buy 1 DVD and 3 CDs?

Answers

Answer:

Step-by-step explanation:

Lets Price of a DVD is fixed i.e. 15

and One CD price is 5 (Not fixed)

In First situation

2 DVDs and 1 CD cost = 35 as given

2 x 15 + 5 = 35

Lets one CD price is 7.5

In Second situation

2 x 15 + 2 x 7.5 = 45

Its mean CD price may be between 5 to 7.5

In asked scenario, Martin has 50

1 DVD and 3 CDs?

1 x 15 + 3 x 7.5 = 37.5

37.5 is lesser than 50

Hence Martin has enough to buy 1 DVD and 3 CDs.

Fill in the blank with a number to make the expression a perfect square.
u^2+8u+?

Answers

Answer:

16

Step-by-step explanation:

Hello, do you remember that result?

For any a and b real numbers,

[tex](a+b)^2=a+2\cdot a \cdot b+b^2[/tex]

In this example, we have.

[tex]u^2+8u=u^2+2\cdot 4 \cdot u\\\\\text{ This is the beginning of ... } u^2+8u+4^2=u^2+8u+16\\\\\text{ So, we need to add 16 to make a perfect square}\\\\u^2+8u+\boxed{16}=u^2+2\cdot 4\cdot u +4^2=(u+4)^2[/tex]

Hope this helps.

Do not hesitate if you need further explanation.

Thank you

There are 9 classes of 25 students each, 4 teachers, and two times as many chaperones as teachers.
Each bus holds a total of 45 people.
What is the least number of buses needed for the field trip?

Answers

5 buses is the answer pls mark me brainliest

Least number of bus require for trip = 5 buses

What is Unitary method?

It is a method where we find the value of a single unit from the value of multiple units and the value of multiple units from the value of a single unit.

Steps to Use Unitary Method

First, let us make a note of the information we have. There are 5 ice-creams. 5 ice-creams cost $125.

Step 1: Let’s find the cost of 1 ice cream. In order to do that, divide the total cost of ice-creams by the total number of ice-creams. The cost of 1 ice-cream = Total cost of ice-creams/Total number of ice-creams = 125/5 = 25. Therefore, the cost of 1 ice cream is $25.

Step 2: To find the cost of 3 ice-creams, multiply the cost of 1 ice cream by the number of ice-creams. The cost of 3 ice-creams is cost of 1 ice-cream × number of ice-creams = 25 × 3 = $75. Finally, we have the cost of 3 ice-creams i.e. $75.

Given:

Total number of classes = 9

Number of student in each class = 25

Number of teacher = 4

Number of chaperones = Double of teacher

Bus hold = 45 people

Now,

Total number of student = 9 × 25

                                         = 225

Number of chaperones = 4 × 2

                                         = 8

Total people = 225 + 8 + 4

                     = 237

Least number of bus require for trip = Total people / Bus hold

= 237 / 45

= 5.266

Learn more about unitary method here:

https://brainly.com/question/22056199

#SPJ2

LCM of x<sup>2</sup>+5x+6 and x<sup>2</sup>-x-6 is ………………………





Answers

Answer:

[tex] (x^2 - 9)(x + 2) [/tex]

Step-by-step explanation:

Given:

[tex] x^2 + 5x + 6 [/tex]

[tex] x^2 - x - 6 [/tex]

Required:

LCM of the polynomials

SOLUTION:

Step 1: Factorise each polynomial

[tex] x^2 + 5x + 6 [/tex]

[tex] x^2 + 3x + 2x + 6 [/tex]

[tex] (x^2 + 3x) + (2x + 6) [/tex]

[tex] x(x + 3) + 2(x + 3) [/tex]

[tex] (x + 2)(x + 3) [/tex]

[tex] x^2 - x - 6 [/tex]

[tex] x^2 - 3x +2x - 6 [/tex]

[tex] x(x - 3) + 2(x - 3) [/tex]

[tex] (x + 2)(x - 3) [/tex]

Step 2: find the product of each factor that is common in both polynomials.

We have the following,

[tex] x^2 + 5x + 6 = (x + 2)(x + 3) [/tex]

[tex] x^2 - x - 6 = (x + 2)(x - 3) [/tex]

The common factors would be: =>

[tex] (x + 2) [/tex] (this is common in both polynomials, so we would take just one of them as a factor.

[tex] (x + 3) [/tex] and,

[tex] (x - 3) [/tex]

Their product = [tex] (x - 3)(x + 3)(x +2) = (x^2 - 9)(x + 2) [/tex]

. Find two polynomial expressions whose quotient, when simplified, is 1/x . Use that division problem to determine whether polynomials are closed under division.

Answers

Answer:

The two polynomials are:

(x + 1) and (x² + x)

Step-by-step explanation:

A polynomial is simply an expression which consists of variables & coefficients involving only the operations of addition, subtraction, multiplication, and non - negative integer exponents of variables.

Now, 1 and x are both polynomials. Thus; 1/x is already a quotient of a polynomial.

Now, to get two polynomial expressions whose quotient, when simplified, is 1/x, we will just multiply the numerator and denominator by the same polynomial to get more quotients.

So,

Let's multiply both numerator and denominator by (x + 1) to get;

(x + 1)/(x(x + 1))

This gives; (x + 1)/(x² + x)

Now, 1 and x are both polynomials but the expression "1/x" is not a polynomial but a quotient and thus polynomials are not closed under division.

Please solve (will make brainiest)

Answers

Answer:

1a) 1/64

1b) 1/169

1c) 1/9

Step-by-step explanation:

You have to apply Indices Law :

[tex] {a}^{ - n} = \frac{1}{ {a}^{n} } [/tex]

Question A,

[tex] {4}^{ - 3} = \frac{1}{ {4}^{3} } = \frac{1}{64} [/tex]

Question B,

[tex] {13}^{ - 2} = \frac{1}{ {13}^{2} } = \frac{1}{169} [/tex]

Question C,

[tex] {( - 3)}^{ - 2} = {( - \frac{1}{3}) }^{2} = \frac{1}{9} [/tex]

Remember, a percent is a fractional part
of 100. In a bag of candy, 15 of the 50
pieces are red. What percentage of the
candy is red?
mex
B 50%.
C 3006
D 659​

Answers

Answer:

Step-by-step explanation:

B

Answer:

The answer would be 30% (although I don't see that as an answer).

Step-by-step explanation:

This is because when you multiply the denominator times a number that makes the denominator 100 and multiply that same number by the numerator you get the percentage of the sample you are looking at on the numerator.

15/50 = (15*2)/(50*2) = 30/100 = 30%

A personal trainer keep track of the number of minutes each of his 20 clients exercise on the treadmill and the number of calories each client burned during that time removing. which TWO of these data points will cause the correlation coefficient to decrease the most?


A). Data point A

B). Data point B

C). Data point C

D). Data point D

Answers

Answer:

Data Point B and Data point E

Step-by-step explanation:

Data point B and data point E are the farthest and are more distant away from the best line of fit compared to other data points. The more clustered data points are, the more the correlation that exists between the variables in question.

Therefore, data point B and data point E, will cause the correlation coefficient to decrease the most.

1. In a right triangle, the lengths of the legs are a and b. Find the length of a hypotenuse, if: a=1, b=1; 2. In a right triangle, the length of a hypotenuse is c and the length of one leg is a. Find the length of the other leg, if: c=5, a=3;

Answers

Answer:

1. [tex]c = \sqrt{2}[/tex].

2. b = 4.

Step-by-step explanation:

To solve these two questions, keep the Pythagorean Theorem in mind: [tex]a^2 + b^2 = c^2[/tex]. Also remember that measurements cannot be negative, so we will disregard the negative answers.

1. a = 1, and b = 1. c = ?

[tex]1^2 + 1^2 = c^2[/tex]

[tex]1 + 1 = c^2\\[/tex]

[tex]c^2 = 1 + 1\\c^2 = 2\\\sqrt{c^2} = \sqrt{2}\\c = \sqrt{2}[/tex]

2. a = 3, c = 5. b = ?

[tex]3^2 + b^2 = 5^2\\9 + b^2 = 25\\b^2 = 16\\\sqrt{b^2} = \sqrt{16}\\b = 4[/tex]

Hope this helps!

1: The best statement for reason 6 of this proof is -∠A ≅ ∠C
-∠B ≅ ∠D
-∠B and ∠D are supplements
-∠B ≅ ∠B

2.The best reason for statements 3.5. and 7 in this proof is
- Alternate interior angles are congruent.
-Corresponding angles are congruent.
-Alternate exterior angles are congruent.
-Interior angles on the same sides of a transversal are supplements.

3. The best statement for reason 8 of this proof is
-∠B ≅ ∠B -∠A and ∠C are supplements.
-∠B ≅ ∠D
-∠A ≅ ∠C

Answers

Answer:

1) -∠B ≅ ∠D

2) -Interior angles on the same side of a transversal are supplementary

3) -∠A ≅ ∠C

Step-by-step explanation:

1) Given that ∠A and ∠B are supplements and ∠A and ∠D are supplements, we have; ∠B ≅ ∠D

2)  Given that ABCD is a parallelogram, therefore ∠A and ∠B,  ∠A and ∠D and ∠B and ∠C  are interior angles on the same side of a transversal and are therefore supplementary

3) Given that ∠A and ∠B and ∠B and ∠C are supplementary, therefore, ∠A ≅ ∠C.

A laundry basket contains 18 blue socks and 24 black socks. What is the probability of randomly picking 2 black socks, with replacement, from the basket?

Answers

Answer:

144/441

Step-by-step explanation:

There are 18+24=42 total socks

There are 24 black socks

So the probability is (24/42)*(24/42)=12/21 * 12/21 = 144/441

Answer:

189

Step-by-step explanation:

An average person's hair grows at a rate of 19cm per year how fast in inches per month does the average person hair grow in conversion factor round you answer to the nearest tenths

Answers

Answer:

Around 1.6 cm per month

Step-by-step explanation:

We can set up a proportion to find how much the hair grows per month. It's important to note that there are 12 months in a year, so we can represent a year as 12 months.

[tex]\frac{19}{12} = \frac{x}{1}[/tex]

We can now cross multiply:

[tex]19\cdot1=19\\\\19\div12=1.58\overline{33}[/tex]

1.58333... rounds to 1.6.

Hope this helped!

Point B is on line segment AC. Given BC=9 and AB=11, determine the length AC.

Answers

Answer:

[tex]AC=20[/tex]

Step-by-step explanation:

The line segment AC is the entire length of the line. Within this segment, point  B is found.

Point B, in a way, splits the segment into two, creating the segments AB and BC.

To find the length of AC, add the lengths of the lines AB and BC together:

[tex]AB=11\\BC=9\\AB+BC=AC\\11+9=AC\\20=AC[/tex]

The length of AC is 20.

:Done

Answer:

20 units.

Step-by-step explanation:

Segment AC is broken into two parts by point B. That means that the length of segment AB plus the length of segment BC equals the length of segment AC.

If BC = 9, and AB = 11, AC = 9 + 11 = 20 units.

Hope this helps!

HELP ASAP PLEASE!!!!!!!!!!!!!!!!!

Answers

Answer:

A

C

D

Step-by-step explanation:

√54 or√9 *√6 or √27 *√4

are equal to the answer.

You can do that by doing the square of outer number which is 3 which equals to 9 when squared and multiplying that with the number inside the square root.

I need help on both answers. They’re different from my other problems so I’m kinda confused

Answers

i think it would be b but
i’m not sure about the other one

Let f(p) be the average number of days a house stays on the market before being sold for price p in $1,000s. Which statement best describes the meaning of f(250)?

Answers

Answer:

Hey There!! The Correct answer C: ) is the average number of days a house stays on the market before being sold for price p in $1,000s

A little more clearer explanation:

p is the price in $1000s, and

f(p) is the number of days before its sold for p

Hence, f(250) would be the number of days before its sold for 250,000 (since p is in $1000s)

Answer choice C is the correct one.

Hope It Helped!~ ♡

ItsNobody~ ☆

Answer: C

Step-by-step explanation:  This is the average number of days the house stayed on the market before being sold for $250,000

|3x–1|=8 please help!!!!!

Answers

Answer: -3

Add 1 to both sides

[tex]3x-1+1=8+1[/tex]

[tex]3x=9[/tex]

Divide both sides by 3

[tex]3x/3=9/3\\x=3[/tex]

What is the rate of change and initial value for the linear relation that includes the points shown in the table?
ху
1 | 20
3 | 10
5 | 0
7 | -10
A. Initial value: 20, rate of change: 10
B. Initial value: 30, rate of change: 10
C. Initial value: 25, rate of change: -5
D. Initial value: 20, rate of change: -10

Answers

Answer:

C, at 0/25, 1/20, 2/15, 3/10,...

Answer:

C

Step-by-step explanation:

At the Olympic games, many events have several rounds of competition. One of these events is the men's 100 100100-meter backstroke. The upper dot plot shows the times (in seconds) of the top 8 88 finishers in the final round of the 2012 20122012 Olympics. The lower dot plot shows the times of the same 8 88 swimmers, but in the semifinal round. Which pieces of information can be gathered from these dot plots? (Remember that lower swim times are faster.) Choose all answers that apply: Choose all answers that apply:

Answers

Answer:

The center of the semifinal round distribution is greater than the center of final round distribution.

The variability in the semifinal round distribution is less than variability in the final round distribution.

Step-by-step explanation:

The mean value of each distribution set is not calculates as the center of semifinal round distribution is greater than the final round distribution. MAD Mean Absolute Deviation is calculated from the dotted graph plot, the distribution of semifinal round is less spread out than the final round distribution.

Answer:

correct answer is None of the above i understood nothing the other person was trying to say...

Step-by-step explanation:

mark me brainliest please...

Type the correct answer in the box. Use numerals instead of words. If necessary, use / for the fraction bar. Stacy goes to the county fair with her friends. The total cost of ride tickets is given by the equation c = 3.5t, where c is the total cost of tickets and t is the number of tickets. If Stacy bought 15 tickets, she would spend $

Answers

Answer:

$52.2

Step-by-step explanation:

Given her total cost of ride tickets modeled by the equation c = 3.5t where c is the total cost of tickets and t is the number of tickets, If Stacy bought 15 tickets, to know the amount she would spend on 15 tickets, we will substitute t = 15 into the modeled equation as shown;

[tex]c = 3.5t\\when t = 15\\\\c = 3.5(15)\\\\c = \frac{35}{10} * 15\\ \\c = \frac{5*7}{5*2} * 15\\\\[/tex]

[tex]c = \frac{7}{2} * 15\\ \\c = \frac{105}{2}\\ \\c = \ 52.2[/tex]

Hence Stacy would spend $52.2 on 15 tickets

Answer:

I hope this helps!

Step-by-step explanation:

Z= -3 - 8i Find the angle θtheta (in degrees) that z makes in the complex plane. Round your answer, if necessary, to the nearest tenth. Express θtheta between -180 180 degrees.

Answers

Answer:  -110.6 degrees approximately

The angle is negative to indicate a clockwise rotation.

======================================================

Explanation:

Z = -3 - 8i is in the form z = a+bi with a = -3 and b = -8

In the complex plane the point (a,b) represents the location of z = a+bi

Define three points with the locations

P = (a,b) = (-3,8)

Q = (0,0)

R = (10,0)

The angle PQR is the angle theta we're looking for. This is the angle formed between the positive x axis and the terminal point (a,b)

Use the arctan function to find theta

theta = arctan(b/a)

theta = arctan( (-8)/(-3) )

theta = 69.4439547804166

theta = 69.4 degrees approximately

Note how this theta value is in quadrant Q1, but (a,b) = (-3, -8) is in Q3

So we need to add 180 degrees to adjust this error.

69.4+180 = 249.4

and we're now in the proper quadrant. We would stop here if your teacher did not put the restriction that theta must be between -180 and 180.

However, this restriction is in place so we need to find the difference of 360 and 249.4 to get 360-249.4 = 110.6

-----------

The angle 249.4 degrees is coterminal to -110.6 degrees. They both point in the same direction.

angle 249.4 degrees is found by starting pointing directly east and rotating 249.4 degrees counterclockwise

angle -110.6 degrees is found by starting directly east and rotating 110.6 degrees clockwise.

Check out the diagram below. I used GeoGebra to make the diagram.

4x
5.
If 7:5 = (x + 2y): (x - y), find the value of
5y

Answers

Answer:

5/2 OR 2.5

Step-by-step explanation:

( x + 2y ) = 7 , ( x - 2y ) = 5

x = 7 - 2y , x = 5 + 2y

substitute the two eqns together:

7 - 2y = 5 + 2y

7 - 5 = 2y + 2y

2 = 4y

y = 1/2

when y = 1/2 ,

5y = 5(1/2)

= 5/2 OR 2.5

James conducted an experiment with 4 possible outcomes. He determined that the experimental probability of event A happening is 10 out of 50. The theoretical probability of event A happening is 1 out of 4. Which action is most likely to cause the experimental probability and theoretical probabilities for each event in the experiment to become closer? removing the last 10 trials from the experimental data completing the experiment many more times and combining the results to the trials already done including a fifth possible outcome performing the experiment again, stopping immediately after each event occurs once

Answers

Answer:

Completing the experiment a few more times and combining the results to the trails already done.

Answer:

Completing the experiment a few more times and combining the results to the trails already done.

Step-by-step explanation:

I answered all my work correctly but I don’t understand this one.

Answers

Take your x values in each coordinate and subtract 2, and take your y values and subtract 1.

Q (0-2), (-1-1)
= (-2,-2)

D (-2-2), (2-1)
= (-4, 1)

V (2-2), (4-1)
= (0,3)

J (3-2), (0-1)
= (1,-1)

You can also draw it on a graph and then translate all coordinates 2 units left and 1 down to see the end results.
Other Questions
Which statements about command economic systems are false? Select all that apply.(Multiple choice btw)Prices are set by the government.Supply and demand are the basis for economic decisions.People's jobs are chosen by the government.Businesses are driven by profit. A pair of narrow, parallel slits separated by 0.230 mm is illuminated by green light ( = 546.1 nm). The interference pattern is observed on a screen 1.50 m away from the plane of the parallel slits.A) Calculate the distance from the central maximum to the first bright region on either side of the central maximum.B) Calculate the distance between the first and second dark bands in the interference pattern. Company A has a patent with a book value of $500,000. At December 31, 2019, the fair value of the patent is determined to be $450,000. The expected future cash flows associated with the patent is $650,000. What amount of impairment should Company A record on this patent at 12/31/19? 13.List 4 safety symbols that would be seen if you are working with a material that is biohazard, such as bacteria. What is health science? Suppose that $2000 is invested at a rate of 2.6% , compounded semiannually. Assuming that no withdrawals are made, find the total amount after 10 years. Write TRUE or FALSE(a)A 'system' is the part of an organism that carries out a certain function. I___ a new bike yesterday? A simple arrangement by means of which e.m.f,s. are compared is known 2. The world has now become a global village in many respects. a) Explain any 5 factors working to make the world a global village for businesses. b) Discuss 4 major reasons why businesses go global. Keith, an employee of Sunbeam, Inc., has gross salary for May of $15,000. The entire amount is under the OASDI limit of $118,500 and thus subject to FICA. He is also subject to federal income tax at a rate of 20%. Which of the following is a part of the journal entry to record the disbursement of his net pay? (Assume a FICAOASDI Tax of 6.2% and FICAMedicare Tax of 1.45%.) (Round the final answer to the nearest dollar.) Select the word that most effectively connects the ideas in this sentence: She didn't mind the long drives to visit her friend, who had moved to the city. _______, she enjoyed the adventure. But For example In fact However Select the correct answer. Which verb form best completes this sentence? Eduardo y Fernando al mercado . fueron B. ful C. How did the Renaissance affect independent morarchs I need help on this question :( Tatham Corporation produces a single product. The standard costs for one unit of its Clan product are as follows: Direct materials (8 pounds at $0.70 per pound) $5.60Direct labor (2 hours at $8 per hour) 16.00 Variable manufacturing overhead (2 hours at $7 per hour) 1,400Total 3,560During November Year 2, 3,500 units of Clan were produced. The costs associated with November operations were as follows: Material purchased (35,000 pounds at $0.80 per pound) 28,000Material used in production (31,500 pounds) Direct labor (7,500 hours at $7.50 per hour) 56,250Variable manufacturing overhead incurred 55,500What is the variable overhead efficiency variance for Clan for November Year 2?1. $3,500 favorable.2. $3,500 unfavorable.3. $4,000 favorable. 4. $4,000 unfavorable. please help :) Write the number shown in standard notation. 6.94 x 10 to the 6 power A. 694 B. 6,940,000 C. 694,000,000 D. 694,000 observe the virual skateboarder coming down the hill and over the ramp describe how each of newtons laws of motion can be observed in this action you can choose the dry wet or muddy conditions or some combination of these Find the value of the expression: mb m^2 for m=3.48 and b=96.52 Suppose that the cost of renting a snowmobile is $37.50 for 5 hours. a. If c represents the cost and represents the hours, which variable is the dependent variable?b. What would be the cost of renting 2 snowmobiles for 5 hours? please and thanks ( i"ll give brainiest )